Answered

Westonci.ca makes finding answers easy, with a community of experts ready to provide you with the information you seek. Join our platform to connect with experts ready to provide detailed answers to your questions in various areas. Our platform provides a seamless experience for finding reliable answers from a network of experienced professionals.

its accounting please help me​

Its Accounting Please Help Me class=

Sagot :

9514 1404 393

Answer:

  1a: $84; 1b: 30,000A +15,000B; 1c: 37,000A +18,500B

  2: 4,000 composite units

Step-by-step explanation:

We assume your "composite unit" consists of the smallest set of units in the ratio in which they are sold.

1)

A "composite unit" is 1B +2A. The contribution margins are the difference between the selling price and the variable cost per unit:

  A: 72 -40 = 32

  B: 48 -28 = 20

  composite unit: 2A+B = 2(32) +20 = 84

(a) The contribution margin per composite unit is $84.

__

(b) The break-even point is the number of composite units that have a total contribution margin equal to the total fixed costs:

  84n = 1260000

  n = 1260000/84 = 15000

30,000 units of A and 15,000 units of B must be sold to break even.

__

(c) The additional number of composite units required to achieve pre-tax income of 294000 is ...

  294,000/84 = 3500

37,000 units of A and 18,500 units of B must be sold for a pretax income of $294,000.

__

2)

Contribution margins per unit are ...

  H: 6 -4 = 2

  C: 8 -6 = 2

  I: 10 -7 = 3

A composite unit is 3H +2C +I, so the contribution margin per composite unit is ...

  3(2) +2(2) +1(3) = 6 +4 +3 = 13 . . . dollars

The break-even point is 52,000/13 = 4,000 composite units